0

For random variables $X$ and $Y$, the conditional entropy of $Y$ given $X$is defined as $$H(Y|X) = - \sum_{x, y} p(x, y) \log p(y|x) = \sum_x p(x) H(Y|X=x)$$ where $$H(Y|X=x) = - \sum_y p(y|x) \log p(y|x)$$

note: $p(x, y) = p(x) \times p(y|x)$ can be used to prove the above equality.

Similarly, for $H(Y|X, Z)$, we write $$H(Y|X, Z) = \sum_z p(z) H(Y|X, Z=z)$$ where $$H(Y|X, Z=z) = - \sum_{x, y} p(x, y|z) \log p(y|x, z)$$

This is Definition 2.15 of chapter II from Information Theory and Network Coding book.

I get confused on how to prove $H(Y|X, Z)$.

Joe
  • 197

1 Answers1

1

This looks simple, if you look at the first proof:

$$H(Y|X) = - \sum_{x, y} p(x, y) \log p(y|x) = - \sum_{x, y} p(x) p(y|x) \log p(y|x).$$

Performing the sum over $y$, we get:

$$H(Y|X) = \sum_x p(x) H(Y|X=x)$$

Similarly, for $H(Y|X,Z)$:

$$H(Y|X, Z) = \sum_{y,x,z} p(y,x,z) \log p(y|x,z) = \sum_{y,x,z} p(z) p(y,x|z) \log p(y|x,z)$$

performing sum over $y,x$:

$$H(Y|X, Z) = \sum_z p(z) H(Y|X, Z=z)$$

Hope this cleared your doubts. If not, let's discuss in this thread.

amWhy
  • 210,739
cyborg_
  • 26
  • Thanks, I would like to understand why $p(y, x, z) = p(z)p(y, x|z)$. Is there a general formula when we have more than $2$ variables? – Joe Mar 25 '20 at 09:47
  • https://math.stackexchange.com/questions/176301/conditional-and-joint-probability-manipulations-when-there-are-3-variables

    check this answer

    – cyborg_ Mar 25 '20 at 13:08
  • Thanks, I think I have a problem with $H(Y|X,Z)$. I faced a problem in proving the following inequality $H(Y|X,Z) \leq H(Y|Z)$. Could you give me a hint, please? – Joe Mar 25 '20 at 16:52
  • 2
    I found a method on how we can prove it. We have,

    (1) $H(Y|X,Z)=H(X, Y|Z)-H(X|Z)$ ([proof](see: https://math.stackexchange.com/q/2676703)) and (2) $I(X;Y|Z) = H(X|Z)-H(X|Y,Z)$.

    From (1), we get $H(X|Y,Z)=H(Y, X|Z)-H(Y|Z)$.

    So $I(X;Y|Z) = H(X|Z)+H(Y|Z)-H(Y, X|Z)$. $H(Y|X,Z)=H(X,Y|Z)-I(X;Y|Z)+H(Y|Z)-H(Y,X|Z)=H(Y|Z)-I(X;Y|Z)\leq H(Y|Z)$.

    Because $H(X,Y|Z)=H(Y,X|Z)$ by symmetry and $I(X;Y|Z)\geq 0$. Am I correct in my proof?

    – Joe Mar 25 '20 at 18:26